How can I integrate by parts this one?

  • Thread starter Thread starter glomar
  • Start date Start date
  • Tags Tags
    Integrate parts
Click For Summary
The integral 1/(sqrt[x] * ln[x]) from 2 to infinity cannot be solved as it does not converge towards a limit at infinity. Attempts to use integration by parts with u = ln[x] and dv = 1/sqrt(x) lead to expressions that do not yield a definitive result. Participants suggest that the original problem may contain a typo, possibly intending to ask about a different integral. It is recommended to calculate definite integrals over finite intervals instead. The consensus is that the integral as stated is not solvable.
glomar
Messages
5
Reaction score
0

Homework Statement



I have to solve the following integral

1/(sqrt[x] * ln[x]) from 2 to infinity

Homework Equations





The Attempt at a Solution



u= ln[x] dv=1/sqrt(x)
du= 1/x v= 2 sqrt(x)


If I do this I get

lim (ln b/ sqrt[x] - 4 sqrt[x] - ln2/sqrt[x] + 4 sqrt[x])

Is this the actual result of the integral? Did I substitute correctly?
Thanks in advance!
 
Physics news on Phys.org
I don't think \int \frac{dx}{\sqrt{x}*ln(x)} has an antiderivative.

Do you mean \int \frac{ln(x)}{\sqrt{x}}dx ?
 
glomar said:

Homework Statement



I have to solve the following integral

1/(sqrt[x] * ln[x]) from 2 to infinity

Homework Equations





The Attempt at a Solution



u= ln[x] dv=1/sqrt(x)
du= 1/x v= 2 sqrt(x)


If I do this I get

lim (ln b/ sqrt[x] - 4 sqrt[x] - ln2/sqrt[x] + 4 sqrt[x])

Is this the actual result of the integral? Did I substitute correctly?
Thanks in advance!

This problem can not be solved because the function is not strictly converging towards infinity. You can calculate definite integrals, f.ex. [2,100 000], [2, 1 000 000] and so on, but there is no way this integral reaches a certain limit as x goes towards infinity. I agree with Random Variable, this seems lik a typo.
 
Probably it was an error from the Professor. I'll check that.
Thanks!
 
Question: A clock's minute hand has length 4 and its hour hand has length 3. What is the distance between the tips at the moment when it is increasing most rapidly?(Putnam Exam Question) Answer: Making assumption that both the hands moves at constant angular velocities, the answer is ## \sqrt{7} .## But don't you think this assumption is somewhat doubtful and wrong?

Similar threads

  • · Replies 14 ·
Replies
14
Views
2K
  • · Replies 7 ·
Replies
7
Views
1K
  • · Replies 105 ·
4
Replies
105
Views
6K
  • · Replies 22 ·
Replies
22
Views
3K
  • · Replies 12 ·
Replies
12
Views
3K
Replies
4
Views
2K
Replies
4
Views
2K
  • · Replies 10 ·
Replies
10
Views
2K
  • · Replies 5 ·
Replies
5
Views
2K
Replies
3
Views
2K